Maping from R2 to R3 torus, finding max min and saddle points

  • Thread starter Thread starter Daveyboy
  • Start date Start date
  • Tags Tags
    Max Points Torus
Click For Summary
The discussion focuses on a mapping from R2 to R3 defined by specific functions, where the user has identified four critical points in the image of the mapping. They are struggling to classify these points as local minima, maxima, or saddle points due to complications with the second derivative test. The user notes that applying this test results in zeros for the second derivatives of two components, which complicates the classification process. They express confusion over obtaining different numbers of minima and maxima across the functions involved. The conversation highlights the challenges of analyzing critical points in multivariable calculus.
Daveyboy
Messages
57
Reaction score
0

Homework Statement


this is from ch9 (functions of several variables)of baby rudin

a,b real b>a>0 define a mapping f=(f1,f2),f3) of R2 into R3 by
f1(s,t)=(b+acos(s))cos(t)
f2(s,t)=(b+acos(s))sin(t)
f2(s,t)=asin(s)
I showed that there are exactly 4 points p in K=image(f) such that

gradf1(f-1(p))=0

I am having trouble finding which of these points are local min, max, and saddle points.


The Attempt at a Solution



The points in question correspond to the touple (s,t) when s=(pi)k and t=(pi)j k,j integers.

the points being +/-b +/-a

Okay great, I do not have any developed criteria for the second derivative test, I have no Hessin matrix to work with. I can develop that in my problem, but that seems rather complicated, I have been trying for a while now, and am having trouble.
 
Physics news on Phys.org
after applying the second derivative test to each component of f I find that at each point of the second derivative of f2 and f3 that the value is zero. So I have zeros at all point of the partial derivatives which is bad because I should have 2 saddle points 1 max and one min. Even more confusing is that in f1 I get two min and two max, no saddle points...
Any ideas
 
Question: A clock's minute hand has length 4 and its hour hand has length 3. What is the distance between the tips at the moment when it is increasing most rapidly?(Putnam Exam Question) Answer: Making assumption that both the hands moves at constant angular velocities, the answer is ## \sqrt{7} .## But don't you think this assumption is somewhat doubtful and wrong?

Similar threads

Replies
2
Views
2K
Replies
1
Views
2K
  • · Replies 1 ·
Replies
1
Views
1K
  • · Replies 9 ·
Replies
9
Views
6K
  • · Replies 4 ·
Replies
4
Views
2K
Replies
3
Views
14K
  • · Replies 1 ·
Replies
1
Views
4K
  • · Replies 2 ·
Replies
2
Views
1K
Replies
2
Views
1K
  • · Replies 1 ·
Replies
1
Views
3K